Hello, dear friend, you can consult us at any time if you have any questions, add WeChat: daixieit

ECMT6006 Applied Financial Econometrics

2022S1

Midterm Questions

Note: As required for the online exams, the numbers in the numerical questions and the ordering of the answers in the multiple-answer questions are randomized in the actual exams.

The total points of this exam is 38 points. Please attempt all questions.

1    Multiple-answer questions (10pt)

The following 5 questions are worth 2 points each.  There may be multiple correct answers to each question, please select all correct answers to receive full points.

1.  Suppose we have been analyzing a stock return series, and we identified a time series model

that can predict future volatility very well using past returns.

Select the statement(s) that must be true.

(a)  The stock return series is a martingale difference sequence.

(b)  The stock return series is not a martingale difference sequence.

(c)  The stock return series is a mutually independent sequence.

(d)  The stock return series is not a mutually independent sequence. Answer:  (d).

2.  Let P0 ,P1 ,P2 ,P3  be the stock prices in four periods. Suppose there are no dividend payments. For t = 0, 1, 2 and k ≥ 2 being a positive integer, let

• R be the single-period log return from t to t + 1, and Rt+k  be the multi-period log return from t to t + k;

• R be the single-period arithmetic net return from t to t + 1, and Rt+k  be the multi- period arithmetic net return from t to t + k .

Select the correct statement(s).

(a)  R3  = ut(3)=1 Rt(A) .

(b)  R3  = t(3)=1 Rt(A) .

(c)  R3  = 1 +ut(3)=1 (1 + Rt(A)).

(d)  R0(L)3  = u=1 Rt(L) .

(e)  R0(L)3  = t(3)=1 ln(1 + Rt(A)).

(f)  R0(L)3  = t(3)=1 Rt(A) .

Answer:  (c) and (e).

3.  Suppose that we run the following regression model of the monthly stock returns (R) on the dividend yields (D)

Rt+1  = β0 + β1 Dt + β2 Dt 1 + β3 Dt 2 + et+1

and would like to test H0  : β 1  = β2  = β3 .  We want to formulate this null hypothesis in the general matrix form:

lβ(β)1(0)

β(β)3(2)   .

Please select correct pair(s) of R and r .

(a)  R =  「(l)        ,    r =  「(l) .

(b)  R =  「(l)   1    1(1)  ,    r =  「(l) .

(c)  R =  「(l)      1    1(1)  ,    r =  「(l) .

(d)  R = [0    1    1    1] ,    r = [0].  (e)  R = [1     1    1] ,    r = [0].       (f)  R = [0(0)   2(0)    2    2] ,    r = [ ]0(0) . (g)  R = [ 1    1   1(0)] ,    r = [ ]0(0) .      (h)  R = [0(0)   0(1)    1    1] ,    r = [ ]0(0) .

(i)  R = ] ,    r = [ ]0(0) .

Answer:  (f) and (h).

4.  Consider an ARMA model for asset return series:

Rt  = µt + εt ,    µt  = Et 1 (Rt ) = c + ϕRt 1 + θεt 1 ,

where c,ϕ,θ are constant parameters satisfying certain restrictions such that {Rt } is weakly stationary.

Select the correct statement(s).

(a)  Assuming weak stationarity here has a drawback of not being able to characterize the

time-varying conditional volatility of the asset returns.

(b) If {εt } is a GARCH(1,1) process, then {εt(2)} follows an AR(1) process.

(c) If {εt } follows an ARCH process, then {εt } cannot be a martingale difference sequence.

(d)  {εt } can have time-invariant second moment if it follows an autoregressive conditional heteroskedastic process.

(e)  {εt } is a zero-mean white noise process.

Answer:  (d) and (e).

5.  Please select all correct statements about estimation of ARMA models.

(a)  Under an ARMA model specification, the joint density of the data is given as the product

of the marginal densities.

(b) We can use either least squares method or maximum likelihood method to estimate an

ARMA(1,1) model.

(c) We can use either least squares method or maximum likelihood method to estimate an AR(3) model.

(d) When we use the autoregression to estimate an AR(1) model given the data, the OLS estimates of the model parameters will be the same no matter the error process is assumed to follow a normal distribution or a t distribution.

(e) We can obtain the log-likelihood of the data without making any specific assumptions on

the distributions of the variables in the model.

Answer:  (c) and (d).

2    Numerical-answer questions

The following 4 questions are 2 points each.

1.  Suppose today’s price of IBM stock is $130.  Given some information set, we have formed a conditional mean prediction of $125 for the IBM stock price tomorrow.  Suppose there is no dividend payment, what is our prediction of the the arithmetic net return (in percentage term) of the IBM stock from today to tomorrow?

Answer:  Let Pt  = 130 be the today’s price of IBM stock, and t+1  = Et (Pt+1) = 125 is the predicted price of IBM stock tomorrow. Let R be the arithmetic net return of the IBM stock from today to tomorrow, then by definition of R and the conditional constancy of conditional expectations, we have

Et (R) = Et  ( Pt+P(1) Pt ) = Et (PtPt(+1) Pt ) = Et (PtPt(+1)) Pt  = 12510130 = 3.85%.

2.  Consider the following MA(3) process:

Yt  = c + θ1 εt 1 + θ3 εt 3 + εt ,    εt  ∼ WN(0,σ2 ).

What is the second-order autocorrelation of {Yt }?

Answer:  We have

Var(Yt ) = Var(εt ) + Var(θ1 εt 1) + Var(εt 3)

= σ 2 + θ1(2)σ 2 + θ3(2)σ 2  = (1 + θ1(2) + θ3(2))σ2 .

Cov(Yt ,Yt 2) = Cov(c + εt + θ1 εt 1 + θ3 εt 3,  c + εt 2 + θ1 εt 3 + θ3 εt 5) = Cov(θ3 εt 3,θ 1 εt 3) = θ 1 θ3 σ 2 .

The second-order autocorrelation is

γ2          Cov(Yt ,Yt 2)          θ 1 θ3        

γ0                Var(Yt )           1 + θ1(2) + θ3(2) .

3.  Suppose that we have the following weakly stationary ARMA(1,1)-GARCH(1,1) model for index stock returns {Rt }:

Rt  = µ + ϕRt 1 + εt + θεt 1                                                                   (1)

εt  = σt νt

σt(2)  = ω + αεt(2)1 + βσt(2)1 ,

νt |Ft 1  ∼ F(0, 1),

where Ft 1  denotes the information set up to time t 1 and F(0, 1) denotes some distribution

Answer:  The unconditional mean of Rt  can be obtained by taking expectation on both sides of (1) and apply the weak stationarity condition to get:

E(Rt ) = µ + ϕE(Rt 1) = µ + ϕE(Rt )

and then solve

E(Rt ) =

Rt  = µ + ϕRt 1 + εt + θεt 1

εt  = σt νt

σt(2)  = ω + αεt(2)1 + βσt(2)1 ,                                                    (2)

νt |Ft 1  ∼ F(0, 1),

where Ft 1  denotes the information set up to t 1 and F(0, 1) denotes some distribution with

Var(εt ) = E(εt(2)) = E(σt(2)) := 2 ,

which is time invariant given the model is weakly stationary. Taking expectation on both sides of (2) and imposing stationarity condition yields

E(σt(2)) = ω + αE(εt(2)1) + βE(σt(2)1)

⇒  2  = ω + α2 + β2

2              ω      

1 α β .

3    Short-answer questions (20pt)

The first three questions are answered by entering texts in the text boxes in the Canvas Quiz.  The last question is answered by uploading a file via Canvas Assignment.

1.  Based on what you learned about efficient market hypothesis, give an example of a strong-form (but not weak-form or semi strong-form) efficient market.  (2pt)

Marking:  See definition of strong-form efficiency in Patton (2019, p.  73).  In particular, in a strong-form efficient (but not weak or semi strong form) market, any private  information will not help to form a successful trading/investment strategy to make economic profits.

2.  Briefly explain what, in general, the information criteria are, and how to use the information criteria to perform model selection.  (2pt) If you consider both AIC and BIC to select forecast models, which one do you expect to select the more parsimonious model, and why?  (2pt)

Marking:  I want the students to mention that information criteria reward the model fit and penalize the additional parameters added to the model. In particular, they consist of two parts: one part quantifies the model fit by using measures such as MSE, RSS and the smaller this part is the better the model goodness of fit is; the other part is the penalty part which is higher when there are more parameters in the model. (1pt) The best model selected by an information criterion is the one which minimizes the information criterion.  (1pt)

We expect BIC to select the more parsimonious model  (1pt) because it imposes a heavier penalty to each additional parameter than AIC (1pt), or the penalty function of BIC, when plotted against the number of parameters, has a higher slope than that of AIC.

3.  Briefly explain what the out-of-sample analysis is for forecasting model selection.  (2pt) Why is this analysis important comparing to the in-sample model selection methods?  (1pt)

Marking: See the discussion of out-of-sample (OOS) model selection in Section 4.6.2 in Patton (2009, p.117).   In particular, I want the students to mention that in the OOS analysis the sample is split to the estimation period and the pseudo out-of-sample period.  The models are estimated in the estimation period and then the forecasts are produced in the pseudo out-of- sample period. Since the realized data are available during pseudo out-of-sample period, we can evaluate the forecasts of different models by using e.g.  mean squared forecast error (MSFE). The model which provides the lowest MSFE is selected as the best model in the OOS analysis.

As for why OOS analysis is important comparing to in-sample model selection, it is worth men- tioning that forecasting is essentially an out-of-sample problem. The in-sample model selection methods attempt to  predict” how well a model will perform out-of-sample by rewarding in- sample goodness-of-fit and penalizing extra parameters for potentially higher estimation error. But if we can set aside the pseudo out-of-sample period, we may use the actual realized data to directly evaluate the model forecasts by comparing the MSFEs. Note that MSFE incorporates directly a penalty for estimation error and in the OOS analysis we don’t need to impose any ad hoc penalty term.

4.  Consider a two-period model for returns Rt ,t = 1, 2 of an asset. Let e0  = 2, and

Rt  = µ + et ,

et  = σt νt ,

σt  = ^2 + 0.5et(2)1 ,

where µ = 2, and ν1 ,ν2  are independent and identically distributed as

νt  = {  2,   w(w)i(i)t(t)h(h) p(p)r(r)o(o)b(b)a(a)b(b)i(i)l(l)i(i)t(t)y(y) 1(2)/(/)3(3)

for t = 1, 2.  Let Ft  be the information set available at time t.  Please answer the following questions.

(i) What is the probability distribution of R1 ?  (1pt) Answer:  We have

R1  = µ + e1  = µ + σ 1 ν1  = 2 + 2ν1  = 2(1 + ν1 ),

where we used the fact that µ = 2 and σ 1  = ^2 + 0.5e0(2)  = ^2 + 0.5 × 4 = 2.  Given the distribution of ν1 specified in the question, we can deduce that the probability distribution of R1  is simply

R1  = { 2,   w(w).(.)p(p).(.) 1(2)/(/)3(3) w(w)h(h)e(e)n(n) ν(ν)1(1)   ).

(ii) What is the probability distribution of R2 ?  (2pt)

Answer:  Note that

R2  = µ + σ2 ν2  = 2 + ^2 + 0.5e ν1(2) 2  = 2 + ^2 + 0.5σ  ν  ν = 2 + ^2 + 2ν1(2)ν2 .


Given the distribution of ν1  and ν2  specified in the question, we have

^2 + 2ν1(2)  =

and

 2 + 2 × 1 = 4,                            w.p. 2/3 × 2/3 = 4/9,

 

R2  = (

( 2 +^10 × ( 2) = 2 2^10,   w.p.  1/3 × 1/3 = 1/9.

which is the probability distribution of R2 .

(iii)  Compute the conditional mean E1 (R2 ) := E(R2 |F1 ).  (2pt)

Answer:  Note that F1  contains the information of the realization of R1 . If R1  = 4, then

it means ν1  = 1 (with probability 2/3); and if R1  = 2, then it means ν1  = 2 (with


R2 |{R1  = 4} = 2 + 2ν2  = {

Similarly,

R2 |{R1  = 2} = 2 +^10ν2  = {

So,


4,      w.p. 2/3,

2,   w.p.  1/3.

 

2 +^10,     w.p. 2/3,

2 2^10,   w.p.  1/3.


E (R2 |R1  = 4) = 4 × 2/3  2 × 1/3 = 2,                               E (R2 |R1  = 2) = (2 +^10) × 2/3 + (2 2^10) × 1/3 = 2.

Therefore, E(R2 |F1 ) is 2 with probability 1.

(iv)  Compute the conditional variance Var1 (R2 ) = Var(R2 |F1 ).  (2pt)

Answer:   Using the conditional distribution of R2  derived in  (iii), we can compute the conditional variance as follows:

Var(R2 |{R1  = 4}) =  (4 − 2)2 +  ( −2 − 2)2  =  +  = 8,                          Var(R2 |{R1  = 2}) =  (2 +^10 2)2 +  (2 2^10 2)2  =  +  = 20,

 


which implies

Var1 (R2 ) = Var(R2 |F1 ) = 

 

w.p. 2/3

w.p.  1/3.


(v) What are the one-period ahead return point forecasts Rˆt+1 at t = 0 and t = 1, respectively? (2pt)

Answer:  The one-period ahead return point forecasts are given by the conditional expec- tations. Therefore, we have

Rˆ1  = E(R1 |F0 ) = 4 × 2/3 2 × 1/3 = 2,

Rˆ2  = E(R2 |F1 ) = 2     which is independent of R1 .

(vi)  At time t = 1, what is the two-standard-deviation, one-period ahead return interval fore-

cast?  (2pt)

Answer:  The two-standard-deviation one-period ahead interval forecast at time t = 1 is Rˆ2 ± 2^Var1 (R2 )

From the results derived in (iv) and (v), this interval forecast is

[2  2^8,    2 + 2^8]   if R1  = 4,

[2 2^20,    1 + 2^20]   if R1  = 2.